Difference between revisions of "Mathematics/Calculus/Corner cases"
From Thalesians Wiki
< Mathematics | Calculus
(Basic structure.) |
|||
(12 intermediate revisions by the same user not shown) | |||
Line 1: | Line 1: | ||
= Derivatives = | = Derivatives = | ||
== The derivative of <math>\frac{d}{dx} x^x</math> == | |||
=== Question === | |||
What is <math>\frac{d}{dx} x^x</math>? | |||
=== Solution 1 === | |||
* Let <math>y = x^x</math>. | |||
* Take <math>\ln</math> of both sides: <math>\ln y = x \ln x</math>. | |||
* Differentiate both sides: <math>\frac{d}{dx} \ln y = \frac{d}{dx} x \ln x</math>. | |||
* Apply the chain rule on the left-hand side: <math>\frac{d}{dx} \ln y = \frac{1}{y} \cdot \frac{dy}{dx}</math>. | |||
* Apply the product rule on the right-hand side: <math>\frac{d}{dx} x \ln x = 1 \cdot \ln x + x \cdot \frac{1}{x} = \ln x + 1</math>. | |||
* Putting it together, we have <math>\frac{1}{y} \cdot \frac{dy}{dx} = \ln x + 1</math>. | |||
* Hence <math>\frac{dy}{dx} = y (\ln x + 1) = x^x (\ln x + 1)</math>. | |||
=== Solution 2 === | |||
* Note that <math>x = e^{\ln x}</math>, so <math>x^x = (e^{\ln x})^x = e^{x \ln x} </math>. | |||
* Applying the chain rule, <math>\frac{d}{dx} x^x = \frac{d}{dx} e^{x \ln x} = e^{x \ln x} \frac{d}{dx} x \ln x</math>. | |||
* Applying the product rule, <math>\frac{d}{dx} x \ln x = 1 \cdot \ln x + x \cdot \frac{1}{x} = \ln x + 1</math>. | |||
* Therefore <math>\frac{d}{dx} x^x = e^{x \ln x} (\ln x + 1) = x^x (\ln x + 1)</math>. | |||
= Integrals = | = Integrals = | ||
== The integral <math>\int x^x \, dx</math> == | |||
=== Question === | |||
What is <math>\int x^x \, dx</math>? | |||
=== Solution === | |||
* We can write <math>x^x</math> as <math>(e^{\ln x})^x = e^{x \ln x}</math>. | |||
* Consider the series expansion of <math>e^{x \ln x}</math>: | |||
<math>e^{x \ln x} = 1 + (x \ln x) + \frac{(x \ln x)^2}{2!} + \frac{(x \ln x)^3}{3!} + \ldots + \frac{(x \ln x)^i}{i!} + \ldots = \sum_{i=0}^{\infty} \frac{(x \ln x)^i}{i!}</math>. | |||
* We can interchange the integration and summation (we can recognize this as a special case of the Fubini/Tonelli theorems) and write | |||
<math> | |||
\int x^x \, dx = \int \left( \sum_{i=0}^{\infty} \frac{(x \ln x)^i}{i!} \right) \, dx = \sum_{i=0}^{\infty} \left( \int \frac{(x \ln x)^i}{i!} \, dx \right) = \sum_{i=0}^{\infty} \left( \frac{1}{i!} \int x^i (\ln x)^i \, dx \right). | |||
</math> | |||
= Limits = | = Limits = | ||
== The limit of <math>\lim_{x \rightarrow 0^+} x^x</math> == | |||
=== Question === | |||
What is <math>\lim_{x \rightarrow 0^+} x^x</math>? | |||
=== Solution === | |||
* Note that <math>x = e^{\ln x}</math>, so <math>x^x = (e^{\ln x})^x = e^{x \ln x}</math>. | |||
* We can further rewrite this as <math>x^x = e^{x \ln x} = e^{\frac{\ln x}{\frac{1}{x}}}</math>. | |||
* As long as <math>f</math> is continuous and the limit of <math>g</math> exists at the point in question, the limit will commute with composition: | |||
<math> | |||
\lim_{x \rightarrow t} f(g(x)) = f(\lim_{x \rightarrow t} g(x)). | |||
</math> | |||
In our case, <math>e(\cdot)</math> is continuous, so | |||
<math> | |||
\lim_{x \rightarrow 0^+} x^x = e^{\lim_{x \rightarrow 0^+} \frac{\ln x}{\frac{1}{x}}}. | |||
</math> | |||
* The question, then, is what is <math>\lim_{x \rightarrow 0^+} \frac{\ln x}{\frac{1}{x}}</math>. | |||
* As <math>x \rightarrow 0^+</math>, <math>\ln x \rightarrow -\infty</math>, <math>\frac{1}{x} \rightarrow +\infty</math>. In this situation we can apply l'Hôpital's rule: | |||
<math> | |||
\lim_{x \rightarrow 0^+} \frac{\ln x}{\frac{1}{x}} = \lim_{x \rightarrow 0^+} \frac{\frac{d}{dx} \ln x}{\frac{d}{dx} \frac{1}{x}} = \frac{\frac{1}{x}}{-\frac{1}{x^2}} = \frac{\frac{1}{x} \cdot x^2}{-\frac{1}{x^2} \cdot x^2} = -x. | |||
</math> | |||
* Hence <math>\lim_{x \rightarrow 0^+} x^x = e^0 = 1</math>. | |||
== The limits of <math>\lim_{x \rightarrow +\infty} x \sin \frac{1}{x}</math> and <math>\lim_{x \rightarrow -\infty} x \sin \frac{1}{x}</math> == | |||
=== Question === | |||
What are <math>\lim_{x \rightarrow +\infty} x \sin \frac{1}{x}</math> and <math>\lim_{x \rightarrow -\infty} x \sin \frac{1}{x}</math>? | |||
=== Solution === | |||
* Let us rewrite <math>x \sin \frac{1}{x}</math> as <math>\frac{\sin \frac{1}{x}}{\frac{1}{x}}</math>. | |||
* As <math>x \rightarrow +\infty</math>, <math>\frac{1}{x} \rightarrow 0</math> and <math>x \sin \frac{1}{x} \rightarrow 0</math>. | |||
* We have "<math>\frac{0}{0}</math>", so we can apply l'Hôpital's rule. | |||
* Differentiating the numerator in <math>\frac{\sin \frac{1}{x}}{\frac{1}{x}}</math>, we obtain <math>\left(\cos \frac{1}{x}\right) \left(-\frac{1}{x^2}\right)</math>. | |||
* Differentiating the denominator in <math>\frac{\sin \frac{1}{x}}{\frac{1}{x}}</math>, we obtain <math>-\frac{1}{x^2}</math>. | |||
* Thus | |||
<math> | |||
\lim_{x \rightarrow +\infty} x \sin \frac{1}{x} = \lim_{x \rightarrow +\infty} \frac{\sin \frac{1}{x}}{\frac{1}{x}} = \lim_{x \rightarrow +\infty} \frac{\left(\cos \frac{1}{x}\right) \left(-\frac{1}{x^2}\right)}{-\frac{1}{x^2}} = \lim_{x \rightarrow +\infty} \cos \frac{1}{x} = 1. | |||
</math> | |||
* Similarly we can find that <math>\lim_{x \rightarrow -\infty} x \sin \frac{1}{x} = 1</math>. |
Latest revision as of 09:53, 22 December 2020
Derivatives
The derivative of
Question
What is ?
Solution 1
- Let .
- Take of both sides: .
- Differentiate both sides: .
- Apply the chain rule on the left-hand side: .
- Apply the product rule on the right-hand side: .
- Putting it together, we have .
- Hence .
Solution 2
- Note that , so .
- Applying the chain rule, .
- Applying the product rule, .
- Therefore .
Integrals
The integral
Question
What is ?
Solution
- We can write as .
- Consider the series expansion of :
.
- We can interchange the integration and summation (we can recognize this as a special case of the Fubini/Tonelli theorems) and write
Limits
The limit of
Question
What is ?
Solution
- Note that , so .
- We can further rewrite this as .
- As long as is continuous and the limit of exists at the point in question, the limit will commute with composition:
In our case, is continuous, so
- The question, then, is what is .
- As , , . In this situation we can apply l'Hôpital's rule:
- Hence .
The limits of and
Question
What are and ?
Solution
- Let us rewrite as .
- As , and .
- We have "", so we can apply l'Hôpital's rule.
- Differentiating the numerator in , we obtain .
- Differentiating the denominator in , we obtain .
- Thus
- Similarly we can find that .